LSAT and Law School Admissions Forum

Get expert LSAT preparation and law school admissions advice from PowerScore Test Preparation.

User avatar
 Dave Killoran
PowerScore Staff
  • PowerScore Staff
  • Posts: 5850
  • Joined: Mar 25, 2011
|
#72705
This game is also discussed in our Podcast, at the 24:24 mark: LSAT Podcast Episode 37: The November 2019 LSAT Logic Games Section

Complete Question Explanation

(The complete setup for this game can be found here: https://forum.powerscore.com/lsat/viewtopic.php?t=31745)

The correct answer choice is (B).

This is a Rule Substitution question that asks you to suspend the second rule and replace it with a new rule that has the same exact effect. The goal with these question is to mimic the effect of the rule while not adding in any new effects or consequences (or falling short of the exact effect of the original rule).

Since there's no way to predict what the answer choices will contain, you should move immediately to the answers in these questions, assuming you completely understand the rule in question. If you don't know the rule well or aren't 100% comfortable with it, take a moment to review how the rule works and which variables it effects.

In this case, the presence of J knocks out Y, which has the further effect of forcing P to be used as an ingredient.


Answer choice (A): This relationship can be stated as P :arrow: J (which will turn out to be the reverse of what is needed, as seen in answer choice (B)). This answer fails because it would allow J and Y to appear together in some solutions.

Answer choice (B): This is the correct answer. This is a Rule-to-Rule connection answer. We know the rule being replaced is J :dblline: Y, and we also know that P :dblline: Y. So, by adding a rule that states J :arrow: P, we get the following "new" chain:

  • J :arrow: P :dblline: Y

    which results in the original relationship being replaced:

    J :dblline: Y
Since this answer mirrors the effects of the original rule and does not add any new unwanted effects, it is the correct answer.

Answer choice (C): This rule is stated as J :arrow: Y, which actually means that when one of the two is absent, then the other must occur. That would create new scenarios under both basic templates, which is not allowed per the question stem. Thus, this answer choice is incorrect.

Answer choice (D): This answer works in part. When there are exactly 5 ingredients, the group would appear exactly the same (OGKT and then the choice of P or Y), so that matches our original rule. However, when there are 6 ingredients, there are new possibilities (namely that J and Y could appear together) and that eliminates this answer choice.

Answer choice (E): This answer would allow for J and Y to appear together in a 6 ingredient recipe, which does not match the effects the original rule. Thus, this answer is incorrect.
 cecilia
  • Posts: 66
  • Joined: Nov 07, 2011
|
#72868
I lucked out in getting to the right answer almost immediately for this. But just wanted to make sure that ,my reasoning was correct in ruling out (D.) which basically says if 5--> ~J.

So yes, while this situation holds true for the existing set of rules and all previous scenarios for the game, it does NOT force the 2nd condition of the game (the 2nd rule of "J---> ~ Y" ).

So when I went to try and set up this new rule, all i was left staring at was this: O G K/t T/k Y/P
This is consistent with all previous work on the game, but it doesn't actually force the old 2nd rule of "J--> ~Y" to be true. So that's why I ruled it out. It's too, in a sense, too lenient. It doesn't force the J--> ~ Y condition to be true.

Have I got that reasoning correct??
User avatar
 Dave Killoran
PowerScore Staff
  • PowerScore Staff
  • Posts: 5850
  • Joined: Mar 25, 2011
|
#72892
cecilia wrote:I lucked out in getting to the right answer almost immediately for this. But just wanted to make sure that ,my reasoning was correct in ruling out (D.) which basically says if 5--> ~J.

So yes, while this situation holds true for the existing set of rules and all previous scenarios for the game, it does NOT force the 2nd condition of the game (the 2nd rule of "J---> ~ Y" ).

So when I went to try and set up this new rule, all i was left staring at was this: O G K/t T/k Y/P
This is consistent with all previous work on the game, but it doesn't actually force the old 2nd rule of "J--> ~Y" to be true. So that's why I ruled it out. It's too, in a sense, too lenient. It doesn't force the J--> ~ Y condition to be true.

Have I got that reasoning correct??
Yes, excellent job!!
 mostofthetime
  • Posts: 8
  • Joined: Dec 09, 2020
|
#83144
I'm having trouble following your summary of the implications of rules two and three.

When you write, "In this case, the presence of J knocks out Y, which has the further effect of forcing P to be used as an ingredient."

Does that mean you diagram rule three as

if ~y -> p
?
I diagrammed it as

if p -> ~y
and the contrapositive: if y -> ~p
Thank you!
User avatar
 Dave Killoran
PowerScore Staff
  • PowerScore Staff
  • Posts: 5850
  • Joined: Mar 25, 2011
|
#83150
Hi Most,

It's both actually. As detailed here: viewtopic.php?t=31745, you need exactly one of P and Y at all times. So, having one knocks the other out, and not having one forces the other in.

Thanks!
User avatar
 crazydonutperson
  • Posts: 2
  • Joined: Mar 21, 2022
|
#96580
Why is J :dblline: Y represented as such? I know they can't be in together, but they can be in the out group together. Why are we able to use the biconditional?
User avatar
 crazydonutperson
  • Posts: 2
  • Joined: Mar 21, 2022
|
#96581
Never mind... opened my LG bible & found that the double-not arrow only precludes one situation: that where the two terms occur together. Neither of them occurring is still a valid outcome even w/ the double-not. I blame it on covid brain :)

Get the most out of your LSAT Prep Plus subscription.

Analyze and track your performance with our Testing and Analytics Package.